CAT Exam  >  CAT Tests  >  CAT Practice Test - 12 - CAT MCQ

CAT Practice Test - 12 - CAT MCQ


Test Description

30 Questions MCQ Test - CAT Practice Test - 12

CAT Practice Test - 12 for CAT 2024 is part of CAT preparation. The CAT Practice Test - 12 questions and answers have been prepared according to the CAT exam syllabus.The CAT Practice Test - 12 MCQs are made for CAT 2024 Exam. Find important definitions, questions, notes, meanings, examples, exercises, MCQs and online tests for CAT Practice Test - 12 below.
Solutions of CAT Practice Test - 12 questions in English are available as part of our course for CAT & CAT Practice Test - 12 solutions in Hindi for CAT course. Download more important topics, notes, lectures and mock test series for CAT Exam by signing up for free. Attempt CAT Practice Test - 12 | 75 questions in 120 minutes | Mock test for CAT preparation | Free important questions MCQ to study for CAT Exam | Download free PDF with solutions
CAT Practice Test - 12 - Question 1

DIRECTIONS for the question: Read the passage and answer the question based on it.

In the technological economy of the twenty-first century, growth and prosperity are the consequences of a virtuous cycle between innovation and demand. Innovation is how we solve problems and raise living standards, while consumer demand is how markets distribute and incentivize innovation. It is social, civic, and economic inclusion—the full, robust participation of as many people as possible—that drives both innovation and demand. And inclusion requires policies that secure a thriving middle class.

The trickle-down theory—the one that lionizes the rich as “job creators”—insists that the American middle class is a consequence of growth, and that only if and when we have growth can we afford to include more people in our economy. But trickle-down has it exactly backwards: Properly understood, the middle class is the source of all growth and prosperity in a modern technological economy, and economic security is the essential feature of what it means to be included in the middle class.

Economic security is what frees us from the fear that one job loss, one illness—one economic downturn amidst a business cycle guaranteed to produce economic downturns—could cost us our home, our car, our family, and our social status. It’s what grants us permission to invest in ourselves and in our children, and to purchase the non-subsistence goods and experiences that make our lives healthier, happier, and more fulfilling. It gives us the confidence to live our lives with the realistic expectation of a more prosperous and stable economic future, and to take the entrepreneurial risks that are the lifeblood of a vibrant market economy. A secure middle class is the cause of growth, not its effect; in fact, our economy cannot reach its full potential without it. And a middle class that lives in constant fear of falling out of the middle class isn’t truly middle class at all.

From 1950 through 1980, during the heyday of the Great American Middle Class, a combination of New Deal programs, a corporate culture of civic responsibility, and a powerful labor movement provided a majority of American workers with health insurance, unemployment insurance, workers’ compensation insurance, pensions, job security, rising wages, overtime pay, paid vacation, paid sick days, a 40-hour workweek, and access to affordable, high-quality education. These are the benefits that provide the economic security of a decent and dignified life that defines what it means to be middle class, and that led to an unprecedented increase in living standards and economic growth. And under the old economy, they were, and still are, largely provided by one’s employer.

But in transforming the traditional relationship between employer and employee, the new economy is quickly stripping away these benefits. That is why it is essential that we imagine and adopt new policies that guarantee all workers the basic level of economic security necessary to sustain and grow the American middle class, and with it, the economy as a whole. We must acknowledge the radically different needs of a new generation of Americans—many of whom already have more employers in a week than their parents had in a lifetime—by adopting a new “Shared Security System” designed to fit the flexible employment relationships of the “sharing economy.”

Q. It can be inferred from the passage that the author of the passage would agree with all of the statements except:

Detailed Solution for CAT Practice Test - 12 - Question 1

Option 3 is incorrect. The view expressed in option 3 is of the ‘trickle-down theory’, something the author does not agree with.

Option 1 can be derived from the lines: A secure middle class is the cause of growth, not its effect; in fact, our economy cannot reach its full potential without it.

Option 2 can be derived from the lines: From 1950 through 1980, during the heyday of the Great American Middle Class, a combination of New Deal programs, a corporate culture of civic responsibility, and a powerful labor movement provided a majority of American workers with health insurance, unemployment insurance, workers’ compensation insurance, pensions, job security, rising wages, overtime pay, paid vacation, paid sick days, a 40-hour workweek, and access to affordable, high-quality education.

Option 4 can be derived from the lines: But in transforming the traditional relationship between employer and employee, the new economy is quickly stripping away these benefits. That is why it is essential that we imagine and adopt new policies that guarantee all workers the basic level of economic security necessary to sustain and grow the American middle class, and with it, the economy as a whole.

CAT Practice Test - 12 - Question 2

DIRECTIONS for the question: Read the passage and answer the question based on it.

In the technological economy of the twenty-first century, growth and prosperity are the consequences of a virtuous cycle between innovation and demand. Innovation is how we solve problems and raise living standards, while consumer demand is how markets distribute and incentivize innovation. It is social, civic, and economic inclusion—the full, robust participation of as many people as possible—that drives both innovation and demand. And inclusion requires policies that secure a thriving middle class.

The trickle-down theory—the one that lionizes the rich as “job creators”—insists that the American middle class is a consequence of growth, and that only if and when we have growth can we afford to include more people in our economy. But trickle-down has it exactly backwards: Properly understood, the middle class is the source of all growth and prosperity in a modern technological economy, and economic security is the essential feature of what it means to be included in the middle class.

Economic security is what frees us from the fear that one job loss, one illness—one economic downturn amidst a business cycle guaranteed to produce economic downturns—could cost us our home, our car, our family, and our social status. It’s what grants us permission to invest in ourselves and in our children, and to purchase the non-subsistence goods and experiences that make our lives healthier, happier, and more fulfilling. It gives us the confidence to live our lives with the realistic expectation of a more prosperous and stable economic future, and to take the entrepreneurial risks that are the lifeblood of a vibrant market economy. A secure middle class is the cause of growth, not its effect; in fact, our economy cannot reach its full potential without it. And a middle class that lives in constant fear of falling out of the middle class isn’t truly middle class at all.

From 1950 through 1980, during the heyday of the Great American Middle Class, a combination of New Deal programs, a corporate culture of civic responsibility, and a powerful labor movement provided a majority of American workers with health insurance, unemployment insurance, workers’ compensation insurance, pensions, job security, rising wages, overtime pay, paid vacation, paid sick days, a 40-hour workweek, and access to affordable, high-quality education. These are the benefits that provide the economic security of a decent and dignified life that defines what it means to be middle class, and that led to an unprecedented increase in living standards and economic growth. And under the old economy, they were, and still are, largely provided by one’s employer.

But in transforming the traditional relationship between employer and employee, the new economy is quickly stripping away these benefits. That is why it is essential that we imagine and adopt new policies that guarantee all workers the basic level of economic security necessary to sustain and grow the American middle class, and with it, the economy as a whole. We must acknowledge the radically different needs of a new generation of Americans—many of whom already have more employers in a week than their parents had in a lifetime—by adopting a new “Shared Security System” designed to fit the flexible employment relationships of the “sharing economy.”

Q. As per the information given in the passage, it can be deduced that:

I. Economic security provides freedom for the fear of job loss.
II. Economic security provides us the ability to fight economic downturns and plan for the future.
III. Economic security enables individuals to take risks and start their own business venture.

Detailed Solution for CAT Practice Test - 12 - Question 2

Statement I can be derived from the lines: Economic security is what frees us from the fear that one job loss.

Statement II can be derived from the lines: one illness—one economic downturn amidst a business cycle guaranteed to produce economic downturns—could cost us our home, our car, our family, and our social status. It’s what grants us permission to invest in ourselves and in our children, and to purchase the non-subsistence goods and experiences that make our lives healthier, happier, and more fulfilling.

Statement III can be derived from the lines: It gives us the confidence to live our lives with the realistic expectation of a more prosperous and stable economic future, and to take the entrepreneurial risks that are the lifeblood of a vibrant market economy.

1 Crore+ students have signed up on EduRev. Have you? Download the App
CAT Practice Test - 12 - Question 3

DIRECTIONS for the question: Read the passage and answer the question based on it.

In the technological economy of the twenty-first century, growth and prosperity are the consequences of a virtuous cycle between innovation and demand. Innovation is how we solve problems and raise living standards, while consumer demand is how markets distribute and incentivize innovation. It is social, civic, and economic inclusion—the full, robust participation of as many people as possible—that drives both innovation and demand. And inclusion requires policies that secure a thriving middle class.

The trickle-down theory—the one that lionizes the rich as “job creators”—insists that the American middle class is a consequence of growth, and that only if and when we have growth can we afford to include more people in our economy. But trickle-down has it exactly backwards: Properly understood, the middle class is the source of all growth and prosperity in a modern technological economy, and economic security is the essential feature of what it means to be included in the middle class.

Economic security is what frees us from the fear that one job loss, one illness—one economic downturn amidst a business cycle guaranteed to produce economic downturns—could cost us our home, our car, our family, and our social status. It’s what grants us permission to invest in ourselves and in our children, and to purchase the non-subsistence goods and experiences that make our lives healthier, happier, and more fulfilling. It gives us the confidence to live our lives with the realistic expectation of a more prosperous and stable economic future, and to take the entrepreneurial risks that are the lifeblood of a vibrant market economy. A secure middle class is the cause of growth, not its effect; in fact, our economy cannot reach its full potential without it. And a middle class that lives in constant fear of falling out of the middle class isn’t truly middle class at all.

From 1950 through 1980, during the heyday of the Great American Middle Class, a combination of New Deal programs, a corporate culture of civic responsibility, and a powerful labor movement provided a majority of American workers with health insurance, unemployment insurance, workers’ compensation insurance, pensions, job security, rising wages, overtime pay, paid vacation, paid sick days, a 40-hour workweek, and access to affordable, high-quality education. These are the benefits that provide the economic security of a decent and dignified life that defines what it means to be middle class, and that led to an unprecedented increase in living standards and economic growth. And under the old economy, they were, and still are, largely provided by one’s employer.

But in transforming the traditional relationship between employer and employee, the new economy is quickly stripping away these benefits. That is why it is essential that we imagine and adopt new policies that guarantee all workers the basic level of economic security necessary to sustain and grow the American middle class, and with it, the economy as a whole. We must acknowledge the radically different needs of a new generation of Americans—many of whom already have more employers in a week than their parents had in a lifetime—by adopting a new “Shared Security System” designed to fit the flexible employment relationships of the “sharing economy.”

Q. According to the author of the passage, innovation and demand are:

Detailed Solution for CAT Practice Test - 12 - Question 3

Refer to the lines: In the technological economy of the twenty-first century, growth and prosperity are the consequences of a virtuous cycle between innovation and demand. Innovation is how we solve problems and raise living standards, while consumer demand is how markets distribute and incentivize innovation. It is social, civic, and economic inclusion—the full, robust participation of as many people as possible—that drives both innovation and demand. And inclusion requires policies that secure a thriving middle class.

►The sentiment of the passage is best captured by option 2.

Option 1, though partially correct, does not capture the exact sentiment of the author of the passage.

Option 3 is ruled out as innovation and demand are not mutually exclusive.
Option 4 is ruled out as innovation and demand are not two sides of the same coin.

CAT Practice Test - 12 - Question 4

DIRECTIONS for the question: Read the passage and answer the question based on it.

In the technological economy of the twenty-first century, growth and prosperity are the consequences of a virtuous cycle between innovation and demand. Innovation is how we solve problems and raise living standards, while consumer demand is how markets distribute and incentivize innovation. It is social, civic, and economic inclusion—the full, robust participation of as many people as possible—that drives both innovation and demand. And inclusion requires policies that secure a thriving middle class.

The trickle-down theory—the one that lionizes the rich as “job creators”—insists that the American middle class is a consequence of growth, and that only if and when we have growth can we afford to include more people in our economy. But trickle-down has it exactly backwards: Properly understood, the middle class is the source of all growth and prosperity in a modern technological economy, and economic security is the essential feature of what it means to be included in the middle class.

Economic security is what frees us from the fear that one job loss, one illness—one economic downturn amidst a business cycle guaranteed to produce economic downturns—could cost us our home, our car, our family, and our social status. It’s what grants us permission to invest in ourselves and in our children, and to purchase the non-subsistence goods and experiences that make our lives healthier, happier, and more fulfilling. It gives us the confidence to live our lives with the realistic expectation of a more prosperous and stable economic future, and to take the entrepreneurial risks that are the lifeblood of a vibrant market economy. A secure middle class is the cause of growth, not its effect; in fact, our economy cannot reach its full potential without it. And a middle class that lives in constant fear of falling out of the middle class isn’t truly middle class at all.

From 1950 through 1980, during the heyday of the Great American Middle Class, a combination of New Deal programs, a corporate culture of civic responsibility, and a powerful labor movement provided a majority of American workers with health insurance, unemployment insurance, workers’ compensation insurance, pensions, job security, rising wages, overtime pay, paid vacation, paid sick days, a 40-hour workweek, and access to affordable, high-quality education. These are the benefits that provide the economic security of a decent and dignified life that defines what it means to be middle class, and that led to an unprecedented increase in living standards and economic growth. And under the old economy, they were, and still are, largely provided by one’s employer.

But in transforming the traditional relationship between employer and employee, the new economy is quickly stripping away these benefits. That is why it is essential that we imagine and adopt new policies that guarantee all workers the basic level of economic security necessary to sustain and grow the American middle class, and with it, the economy as a whole. We must acknowledge the radically different needs of a new generation of Americans—many of whom already have more employers in a week than their parents had in a lifetime—by adopting a new “Shared Security System” designed to fit the flexible employment relationships of the “sharing economy.”

Q. As per the views of the author of the passage, 'the trickle down' theory:

I. Inverts the role of middle class in society.
II. Essentially treats the middle class an outcome of growth
III. Assumes that the middle class lives in a constant fear of failing.

Detailed Solution for CAT Practice Test - 12 - Question 4

Statements I and II can be derived from the passage: But trickle-down has it exactly backwards: Properly understood, the middle class is the source of all growth and prosperity in a modern technological economy, and economic security is the essential feature of what it means to be included in the middle class.

► Statement III is not related to the 'trickle down' theory and the passage does not state that the theory makes any such assumption.

CAT Practice Test - 12 - Question 5

DIRECTIONS for the question: Read the passage below and answer the questions that follow.
The first systems of writing developed and used by the Germanic peoples were runic alphabets. The runes functioned as letters, but they were much more than just letters in the sense in which we today understand the term. Each rune was an ideographic or pictographic symbol of some cosmological principle or power, and to write a rune was to invoke and direct the force for which it stood. Indeed, in every Germanic language, the word “rune” (from Proto-Germanic *runo) means both “letter” and “secret” or “mystery,” and its original meaning, which likely predated the adoption of the runic alphabet, may have been simply “(hushed) message.” 
Each rune had a name that hinted at the philosophical and magical significance of its visual form and the sound for which it stands, which was almost always the first sound of the rune’s name. For example, the T-rune, called *Tiwaz in the Proto-Germanic language, is named after the god Tiwaz (known as Tyr in the Viking Age). Tiwaz was perceived to dwell within the daytime sky, and, accordingly, the visual form of the T-rune is an arrow pointed upward (which surely also hints at the god’s martial role). The T-rune was often carved as a standalone ideograph, apart from the writing of any particular word, as part of spells cast to ensure victory in battle. 
The runic alphabets are called “futharks'' after the first six runes (Fehu, Uruz, Thurisaz, Ansuz, Raidho, Kaunan), in much the same way that the word “alphabet” comes from the names of the first two Hebrew letters (Aleph, Beth). There are three principal futharks: the 24-character Elder Futhark, the first fully-formed runic alphabet, whose development had begun by the first century CE and had been completed before the year 400; the 16-character Younger Futhark, which began to diverge from the Elder Futhark around the beginning of the Viking Age (c. 750 CE) and eventually replaced that older alphabet in Scandinavia; and the 33-character Anglo-Saxon Futhorc, which gradually altered and added to the Elder Futhark in England. On some inscriptions, the twenty-four runes of the Elder Futhark were divided into three ættir (Old Norse, “families”) of eight runes each, but the significance of this division is unfortunately unknown. Runes were traditionally carved into stone, wood, bone, metal, or some similarly hard surface rather than drawn with ink and pen on parchment. 
This explains their sharp, angular form, which was well-suited to the medium. Much of our current knowledge of the meanings the ancient Germanic peoples attributed to the runes comes from the three “Rune Poems,” documents from Iceland, Norway, and England that provide a short stanza about each rune in their respective futharks (the Younger Futhark is treated in the Icelandic and Norwegian Rune Poems, while the Anglo-Saxon Futhorc is discussed in the Old English Rune Poem). While runologists argue over many of the details of the historical origins of runic writing, there is widespread agreement on a general outline. The runes are presumed to have been derived from one of the many Old Italic alphabets in use among the Mediterranean peoples of the first century CE, who lived to the south of the Germanic tribes. Earlier Germanic sacred symbols, such as those preserved in northern European petroglyphs, were also likely influential in the development of the script. The earliest possibly runic inscription is found on the Meldorf brooch, which was manufactured in the north of modern-day Germany around 50 CE. The inscription is highly ambiguous, however, and scholars are divided over whether its letters are runic or Roman. The earliest unambiguous runic inscriptions are found on the Vimose comb from Vimose, Denmark and the Øvre Stabu spearhead from southern Norway, both of which date to approximately 160 CE. The earliest known carving of the entire futhark, in order, is that on the Kylver stone from Gotland, Sweden, which dates to roughly 400 CE. The transmission of writing from southern Europe to northern Europe likely took place via Germanic warbands, the dominant northern European military institution of the period, who would have encountered Italic writing firsthand during campaigns amongst their southerly neighbors. This hypothesis is supported by the association that runes have always had with the god Odin, who, in the Proto-Germanic period, under his original name *Woðanaz, was the divine model of the human warband leader and the invisible patron of the warband’s activities. The Roman historian Tacitus tells us that Odin (“Mercury” in the interpretatio romana) was already established as the dominant god in the pantheons of many of the Germanic tribes by the first century. 
From the perspective of the ancient Germanic peoples themselves, however, the runes came from no source as mundane as an Old Italic alphabet. The runes were never “invented,” but are instead eternal, pre-existent forces that Odin himself discovered by undergoing a tremendous ordeal.

Q. The word “pantheon” in the passage refers to

Detailed Solution for CAT Practice Test - 12 - Question 5

We find the word "pantheon" in the following line of the passage: "The Roman historian Tacitus tells us that Odin (“Mercury? in the interpretatio romana) was already established as the dominant god in the pantheons of many of the Germanic tribes by the first century." Here, clearly, it refers to Odin being the dominant god amongst all gods of the Germanic tribes. So option b is the correct choice.

CAT Practice Test - 12 - Question 6

DIRECTIONS for the question: Read the passage below and answer the questions that follow.
The first systems of writing developed and used by the Germanic peoples were runic alphabets. The runes functioned as letters, but they were much more than just letters in the sense in which we today understand the term. Each rune was an ideographic or pictographic symbol of some cosmological principle or power, and to write a rune was to invoke and direct the force for which it stood. Indeed, in every Germanic language, the word “rune” (from Proto-Germanic *runo) means both “letter” and “secret” or “mystery,” and its original meaning, which likely predated the adoption of the runic alphabet, may have been simply “(hushed) message.” 
Each rune had a name that hinted at the philosophical and magical significance of its visual form and the sound for which it stands, which was almost always the first sound of the rune’s name. For example, the T-rune, called *Tiwaz in the Proto-Germanic language, is named after the god Tiwaz (known as Tyr in the Viking Age). Tiwaz was perceived to dwell within the daytime sky, and, accordingly, the visual form of the T-rune is an arrow pointed upward (which surely also hints at the god’s martial role). The T-rune was often carved as a standalone ideograph, apart from the writing of any particular word, as part of spells cast to ensure victory in battle. 
The runic alphabets are called “futharks'' after the first six runes (Fehu, Uruz, Thurisaz, Ansuz, Raidho, Kaunan), in much the same way that the word “alphabet” comes from the names of the first two Hebrew letters (Aleph, Beth). There are three principal futharks: the 24-character Elder Futhark, the first fully-formed runic alphabet, whose development had begun by the first century CE and had been completed before the year 400; the 16-character Younger Futhark, which began to diverge from the Elder Futhark around the beginning of the Viking Age (c. 750 CE) and eventually replaced that older alphabet in Scandinavia; and the 33-character Anglo-Saxon Futhorc, which gradually altered and added to the Elder Futhark in England. On some inscriptions, the twenty-four runes of the Elder Futhark were divided into three ættir (Old Norse, “families”) of eight runes each, but the significance of this division is unfortunately unknown. Runes were traditionally carved into stone, wood, bone, metal, or some similarly hard surface rather than drawn with ink and pen on parchment. 
This explains their sharp, angular form, which was well-suited to the medium. Much of our current knowledge of the meanings the ancient Germanic peoples attributed to the runes comes from the three “Rune Poems,” documents from Iceland, Norway, and England that provide a short stanza about each rune in their respective futharks (the Younger Futhark is treated in the Icelandic and Norwegian Rune Poems, while the Anglo-Saxon Futhorc is discussed in the Old English Rune Poem). While runologists argue over many of the details of the historical origins of runic writing, there is widespread agreement on a general outline. The runes are presumed to have been derived from one of the many Old Italic alphabets in use among the Mediterranean peoples of the first century CE, who lived to the south of the Germanic tribes. Earlier Germanic sacred symbols, such as those preserved in northern European petroglyphs, were also likely influential in the development of the script. The earliest possibly runic inscription is found on the Meldorf brooch, which was manufactured in the north of modern-day Germany around 50 CE. The inscription is highly ambiguous, however, and scholars are divided over whether its letters are runic or Roman. The earliest unambiguous runic inscriptions are found on the Vimose comb from Vimose, Denmark and the Øvre Stabu spearhead from southern Norway, both of which date to approximately 160 CE. The earliest known carving of the entire futhark, in order, is that on the Kylver stone from Gotland, Sweden, which dates to roughly 400 CE. The transmission of writing from southern Europe to northern Europe likely took place via Germanic warbands, the dominant northern European military institution of the period, who would have encountered Italic writing firsthand during campaigns amongst their southerly neighbors. This hypothesis is supported by the association that runes have always had with the god Odin, who, in the Proto-Germanic period, under his original name *Woðanaz, was the divine model of the human warband leader and the invisible patron of the warband’s activities. The Roman historian Tacitus tells us that Odin (“Mercury” in the interpretatio romana) was already established as the dominant god in the pantheons of many of the Germanic tribes by the first century. 
From the perspective of the ancient Germanic peoples themselves, however, the runes came from no source as mundane as an Old Italic alphabet. The runes were never “invented,” but are instead eternal, pre-existent forces that Odin himself discovered by undergoing a tremendous ordeal.

Q. Which of the following statements is incorrect? 

Detailed Solution for CAT Practice Test - 12 - Question 6

Statement 1 - Unlike the Latin alphabet, which is an essentially utilitarian script, the runes are symbols of some of the most powerful forces in the cosmos. 
From the passage, we know this to be true. Runes functioned as letters, but they were much more than just letters. Each rune was an ideographic or pictographic symbol of some cosmological power. 
Statement 2 - Runic writing was probably first used in southern Europe and was carried north by Germanic tribes. 
Again, this is stated in the passage and is correct. 
Statement 3 - The word “rune” and its meaning was derived from the runic alphabet 
The word “rune” means both “letter” and “secret” and its original meaning predated the adoption of the runic alphabet. Hence statement 3 is incorrect. 
Statement 4 - The first runic alphabets date back to the 1st century CE. 
Indeed, according to the passage, the development of the Elder Futhark had begun by the first century. So this statement is correct.

CAT Practice Test - 12 - Question 7

DIRECTIONS for the question: Read the passage below and answer the questions that follow.
The first systems of writing developed and used by the Germanic peoples were runic alphabets. The runes functioned as letters, but they were much more than just letters in the sense in which we today understand the term. Each rune was an ideographic or pictographic symbol of some cosmological principle or power, and to write a rune was to invoke and direct the force for which it stood. Indeed, in every Germanic language, the word “rune” (from Proto-Germanic *runo) means both “letter” and “secret” or “mystery,” and its original meaning, which likely predated the adoption of the runic alphabet, may have been simply “(hushed) message.” 
Each rune had a name that hinted at the philosophical and magical significance of its visual form and the sound for which it stands, which was almost always the first sound of the rune’s name. For example, the T-rune, called *Tiwaz in the Proto-Germanic language, is named after the god Tiwaz (known as Tyr in the Viking Age). Tiwaz was perceived to dwell within the daytime sky, and, accordingly, the visual form of the T-rune is an arrow pointed upward (which surely also hints at the god’s martial role). The T-rune was often carved as a standalone ideograph, apart from the writing of any particular word, as part of spells cast to ensure victory in battle. 
The runic alphabets are called “futharks'' after the first six runes (Fehu, Uruz, Thurisaz, Ansuz, Raidho, Kaunan), in much the same way that the word “alphabet” comes from the names of the first two Hebrew letters (Aleph, Beth). There are three principal futharks: the 24-character Elder Futhark, the first fully-formed runic alphabet, whose development had begun by the first century CE and had been completed before the year 400; the 16-character Younger Futhark, which began to diverge from the Elder Futhark around the beginning of the Viking Age (c. 750 CE) and eventually replaced that older alphabet in Scandinavia; and the 33-character Anglo-Saxon Futhorc, which gradually altered and added to the Elder Futhark in England. On some inscriptions, the twenty-four runes of the Elder Futhark were divided into three ættir (Old Norse, “families”) of eight runes each, but the significance of this division is unfortunately unknown. Runes were traditionally carved into stone, wood, bone, metal, or some similarly hard surface rather than drawn with ink and pen on parchment. 
This explains their sharp, angular form, which was well-suited to the medium. Much of our current knowledge of the meanings the ancient Germanic peoples attributed to the runes comes from the three “Rune Poems,” documents from Iceland, Norway, and England that provide a short stanza about each rune in their respective futharks (the Younger Futhark is treated in the Icelandic and Norwegian Rune Poems, while the Anglo-Saxon Futhorc is discussed in the Old English Rune Poem). While runologists argue over many of the details of the historical origins of runic writing, there is widespread agreement on a general outline. The runes are presumed to have been derived from one of the many Old Italic alphabets in use among the Mediterranean peoples of the first century CE, who lived to the south of the Germanic tribes. Earlier Germanic sacred symbols, such as those preserved in northern European petroglyphs, were also likely influential in the development of the script. The earliest possibly runic inscription is found on the Meldorf brooch, which was manufactured in the north of modern-day Germany around 50 CE. The inscription is highly ambiguous, however, and scholars are divided over whether its letters are runic or Roman. The earliest unambiguous runic inscriptions are found on the Vimose comb from Vimose, Denmark and the Øvre Stabu spearhead from southern Norway, both of which date to approximately 160 CE. The earliest known carving of the entire futhark, in order, is that on the Kylver stone from Gotland, Sweden, which dates to roughly 400 CE. The transmission of writing from southern Europe to northern Europe likely took place via Germanic warbands, the dominant northern European military institution of the period, who would have encountered Italic writing firsthand during campaigns amongst their southerly neighbors. This hypothesis is supported by the association that runes have always had with the god Odin, who, in the Proto-Germanic period, under his original name *Woðanaz, was the divine model of the human warband leader and the invisible patron of the warband’s activities. The Roman historian Tacitus tells us that Odin (“Mercury” in the interpretatio romana) was already established as the dominant god in the pantheons of many of the Germanic tribes by the first century. 
From the perspective of the ancient Germanic peoples themselves, however, the runes came from no source as mundane as an Old Italic alphabet. The runes were never “invented,” but are instead eternal, pre-existent forces that Odin himself discovered by undergoing a tremendous ordeal.

Q. Which of the following can be inferred from the passage? 
a. Runic script was most likely derived from Old Italic script. 
b. Runes were not used so much as a simple writing system, but rather as magical signs to be used for charms. 
c. In the Proto-Germanic period, the god Tiwaz was associated with war, victory, marriage and the diurnal sky. 
d. The knowledge of the meanings attributed to the runes of the Younger Futhark is derived from the three Rune poems.

Detailed Solution for CAT Practice Test - 12 - Question 7

Statement I - Runic script was most likely derived from Italic script. 
True, The runes are presumed to have been derived from one of the many Old Italic alphabets in use among the Mediterranean peoples of the first century CE, who lived to the south of the Germanic tribes. 
Statement II - Runes were not used so much as a simple writing system, but rather as magical signs to be used for charms. 
False, Runes were used as a writing system. They were also used as magical signs. They were more than letters as we understand them today, not just magical signs. 
Statement III - In the Proto-Germanic period, the god Tiwaz was associated with war, victory, marriage and the diurnal sky. 
True, We understand from the passage that Tiwaz was perceived to dwell within the daytime sky, had a martial role and was associated with victory in battle. 
Statement IV - The knowledge of the meanings attributed to the runes of the Younger Futhark is derived from the three Rune poems. 
False, The Younger Futhark is treated in the Icelandic and Norwegian Rune Poems only, not all three rune poems as suggested in the statement above. So, of the four statements above only I and III can be inferred from the passage.

CAT Practice Test - 12 - Question 8

Instructions: Read the passage below and answer the questions that follow.
The first systems of writing developed and used by the Germanic peoples were runic alphabets. The runes functioned as letters, but they were much more than just letters in the sense in which we today understand the term. Each rune was an ideographic or pictographic symbol of some cosmological principle or power, and to write a rune was to invoke and direct the force for which it stood. Indeed, in every Germanic language, the word “rune” (from Proto-Germanic *runo) means both “letter” and “secret” or “mystery,” and its original meaning, which likely predated the adoption of the runic alphabet, may have been simply “(hushed) message.” 
Each rune had a name that hinted at the philosophical and magical significance of its visual form and the sound for which it stands, which was almost always the first sound of the rune’s name. For example, the T-rune, called *Tiwaz in the Proto-Germanic language, is named after the god Tiwaz (known as Tyr in the Viking Age). Tiwaz was perceived to dwell within the daytime sky, and, accordingly, the visual form of the T-rune is an arrow pointed upward (which surely also hints at the god’s martial role). The T-rune was often carved as a standalone ideograph, apart from the writing of any particular word, as part of spells cast to ensure victory in battle. 
The runic alphabets are called “futharks'' after the first six runes (Fehu, Uruz, Thurisaz, Ansuz, Raidho, Kaunan), in much the same way that the word “alphabet” comes from the names of the first two Hebrew letters (Aleph, Beth). There are three principal futharks: the 24-character Elder Futhark, the first fully-formed runic alphabet, whose development had begun by the first century CE and had been completed before the year 400; the 16-character Younger Futhark, which began to diverge from the Elder Futhark around the beginning of the Viking Age (c. 750 CE) and eventually replaced that older alphabet in Scandinavia; and the 33-character Anglo-Saxon Futhorc, which gradually altered and added to the Elder Futhark in England. On some inscriptions, the twenty-four runes of the Elder Futhark were divided into three ættir (Old Norse, “families”) of eight runes each, but the significance of this division is unfortunately unknown. Runes were traditionally carved into stone, wood, bone, metal, or some similarly hard surface rather than drawn with ink and pen on parchment. 
This explains their sharp, angular form, which was well-suited to the medium. Much of our current knowledge of the meanings the ancient Germanic peoples attributed to the runes comes from the three “Rune Poems,” documents from Iceland, Norway, and England that provide a short stanza about each rune in their respective futharks (the Younger Futhark is treated in the Icelandic and Norwegian Rune Poems, while the Anglo-Saxon Futhorc is discussed in the Old English Rune Poem). While runologists argue over many of the details of the historical origins of runic writing, there is widespread agreement on a general outline. The runes are presumed to have been derived from one of the many Old Italic alphabets in use among the Mediterranean peoples of the first century CE, who lived to the south of the Germanic tribes. Earlier Germanic sacred symbols, such as those preserved in northern European petroglyphs, were also likely influential in the development of the script. The earliest possibly runic inscription is found on the Meldorf brooch, which was manufactured in the north of modern-day Germany around 50 CE. The inscription is highly ambiguous, however, and scholars are divided over whether its letters are runic or Roman. The earliest unambiguous runic inscriptions are found on the Vimose comb from Vimose, Denmark and the Øvre Stabu spearhead from southern Norway, both of which date to approximately 160 CE. The earliest known carving of the entire futhark, in order, is that on the Kylver stone from Gotland, Sweden, which dates to roughly 400 CE. The transmission of writing from southern Europe to northern Europe likely took place via Germanic warbands, the dominant northern European military institution of the period, who would have encountered Italic writing firsthand during campaigns amongst their southerly neighbors. This hypothesis is supported by the association that runes have always had with the god Odin, who, in the Proto-Germanic period, under his original name *Woðanaz, was the divine model of the human warband leader and the invisible patron of the warband’s activities. The Roman historian Tacitus tells us that Odin (“Mercury” in the interpretatio romana) was already established as the dominant god in the pantheons of many of the Germanic tribes by the first century. 
From the perspective of the ancient Germanic peoples themselves, however, the runes came from no source as mundane as an Old Italic alphabet. The runes were never “invented,” but are instead eternal, pre-existent forces that Odin himself discovered by undergoing a tremendous ordeal.

Q. Which of the following cannot be reasonably inferred with regard to the beliefs of the Proto-Germanic people?

Detailed Solution for CAT Practice Test - 12 - Question 8

Statement a - Odin came upon the runes after going through a lot of torment. True, stated in the last couple of paragraphs. 
Statement b - The name of a rune was almost always the first sound of a God’s name. 
False, Each rune had a name which was almost always the first sound of the rune’s name. Though the example of the T rune and Tiwaz is given in the passage, it is not stated as a general rule that the name of a rune is the first sound of a God’s name. It is the first sound of the rune’s name. 
Statement c - The cosmological power represented by a rune was invoked by writing it. True.
Statement d - Proto-German Gods were modeled on humans. 
True, The passage gives the example of Wodanaz, the divine model of the human warband leader. 
So the correct answer choice is option b.

CAT Practice Test - 12 - Question 9

DIRECTIONS for the question: Read the passage and answer the question based on it.

If the man to perpetuate whose memory we have this day raised a statue had been asked on what part of his busy life's work he set the highest value, he would undoubtedly have pointed to his voluminous contributions to theology. In season and out of season, he was the steadfast champion of that hypothesis respecting the Divine nature which is termed Unitarianism by its friends and Socinianism by its foes. Regardless of odds, he was ready to do battle with all comers in that cause; and if no adversaries entered the lists, he would sally forth to seek them.

To this, his highest ideal of duty, Joseph Priestley sacrificed the vulgar prizes of life, which, assuredly, were within easy reach of a man of his singular energy and varied abilities. For this object he put aside, as of secondary importance, those scientific investigations which he loved so well, and in which he showed himself so competent to enlarge the boundaries of natural knowledge and to win fame. In this cause he not only cheerfully suffered obloquy from the bigoted and the unthinking, and came within sight of martyrdom; but bore with that which is much harder to be borne than all these, the unfeigned astonishment and hardly disguised contempt of a brilliant society, composed of men whose sympathy and esteem must have been most dear to him, and to whom it was simply incomprehensible that a philosopher should seriously occupy himself with any form of Christianity.

It appears to me that the man who, setting before himself such an ideal of life, acted up to it consistently, is worthy of the deepest respect, whatever opinion may be entertained as to the real value of the tenets which he so zealously propagated and defended. But I am sure that I speak not only for myself, but for all this assemblage, when I say that our purpose to-day is to do honour, not to Priestley, the Unitarian divine, but to Priestley, the fearless defender of rational freedom in thought and in action: to Priestley, the philosophic thinker; to that Priestley who held a foremost place among "the swift runners who hand over the lamp of life," and transmit from one generation to another the fire kindled, in the childhood of the world, at the Promethean altar of Science.

The main incidents of Priestley's life are so well known that I need dwell upon them at no great length. Born in 1733, at Fieldhead, near Leeds, and brought up among Calvinists of the straitest orthodoxy, the boy's striking natural ability led to his being devoted to the profession of a minister of religion; and, in 1752, he was sent to the Dissenting Academy at Daventry--an institution which authority left undisturbed, though its existence contravened the law. The teachers under whose instruction and influence the young man came at Daventry, carried out to the letter the injunction to "try all things: hold fast that which is good," and encouraged the discussion of every imaginable proposition with complete freedom, the leading professors taking opposite sides; a discipline which, admirable as it may be from a purely scientific point of view, would seem to be calculated to make acute, rather than sound, divines. Priestley tells us, in his "Autobiography," that he generally found himself on the unorthodox side: and, as he grew older, and his faculties attained their maturity, this native tendency towards heterodoxy grew with his growth and strengthened with his strength. He passed from Calvinism to Arianism; and finally, in middle life, landed in that very broad form of Unitarianism by which his craving after a credible and consistent theory of things was satisfied.

Q. What can be inferred from the passage about the early life of Priestly?

Detailed Solution for CAT Practice Test - 12 - Question 9

► Option 1 is incorrect on two counts: ''incongruous'' does not fit in the given context and we cannot say that Priestley''s early life had a significant impact on him.

► Option 2 is again rejected as it is too strong an option.

► Option 3 is correct as it aptly mentions that his childhood experience had a contributory role, something which cannot be denied.

► Option 4 is incorrect as the passage does not mention that Priestley took up any dedicated early work in unorthodox ways

CAT Practice Test - 12 - Question 10

DIRECTIONS for the question: Read the passage and answer the question based on it.

If the man to perpetuate whose memory we have this day raised a statue had been asked on what part of his busy life's work he set the highest value, he would undoubtedly have pointed to his voluminous contributions to theology. In season and out of season, he was the steadfast champion of that hypothesis respecting the Divine nature which is termed Unitarianism by its friends and Socinianism by its foes. Regardless of odds, he was ready to do battle with all comers in that cause; and if no adversaries entered the lists, he would sally forth to seek them.

To this, his highest ideal of duty, Joseph Priestley sacrificed the vulgar prizes of life, which, assuredly, were within easy reach of a man of his singular energy and varied abilities. For this object he put aside, as of secondary importance, those scientific investigations which he loved so well, and in which he showed himself so competent to enlarge the boundaries of natural knowledge and to win fame. In this cause he not only cheerfully suffered obloquy from the bigoted and the unthinking, and came within sight of martyrdom; but bore with that which is much harder to be borne than all these, the unfeigned astonishment and hardly disguised contempt of a brilliant society, composed of men whose sympathy and esteem must have been most dear to him, and to whom it was simply incomprehensible that a philosopher should seriously occupy himself with any form of Christianity.

It appears to me that the man who, setting before himself such an ideal of life, acted up to it consistently, is worthy of the deepest respect, whatever opinion may be entertained as to the real value of the tenets which he so zealously propagated and defended. But I am sure that I speak not only for myself, but for all this assemblage, when I say that our purpose to-day is to do honour, not to Priestley, the Unitarian divine, but to Priestley, the fearless defender of rational freedom in thought and in action: to Priestley, the philosophic thinker; to that Priestley who held a foremost place among "the swift runners who hand over the lamp of life," and transmit from one generation to another the fire kindled, in the childhood of the world, at the Promethean altar of Science.

The main incidents of Priestley's life are so well known that I need dwell upon them at no great length. Born in 1733, at Fieldhead, near Leeds, and brought up among Calvinists of the straitest orthodoxy, the boy's striking natural ability led to his being devoted to the profession of a minister of religion; and, in 1752, he was sent to the Dissenting Academy at Daventry--an institution which authority left undisturbed, though its existence contravened the law. The teachers under whose instruction and influence the young man came at Daventry, carried out to the letter the injunction to "try all things: hold fast that which is good," and encouraged the discussion of every imaginable proposition with complete freedom, the leading professors taking opposite sides; a discipline which, admirable as it may be from a purely scientific point of view, would seem to be calculated to make acute, rather than sound, divines. Priestley tells us, in his "Autobiography," that he generally found himself on the unorthodox side: and, as he grew older, and his faculties attained their maturity, this native tendency towards heterodoxy grew with his growth and strengthened with his strength. He passed from Calvinism to Arianism; and finally, in middle life, landed in that very broad form of Unitarianism by which his craving after a credible and consistent theory of things was satisfied.

Q. The author of the passage would agree with the statement:

Detailed Solution for CAT Practice Test - 12 - Question 10

Refer to the following lines

►"In season and out of season, he was the steadfast champion of that hypothesis respecting the Divine nature which is termed Unitarianism by its friends and Socinianism by its foes. Regardless of odds, he was ready to do battle with all comers in that cause; and if no adversaries entered the lists, he would sally forth to seek them."

►From these lines, it can clearly be seen that one thing that can be said about the work of Priestley is that it was widely debated (it had its supporters and its opponents).

►Considering thing, we find option 4 the apt answer for the given question. None of the other options can be conclusively proven in the given context. Option 1 essentially means that his views were objective (which means factual) in nature. This is something we cannot conclude in the given context.

CAT Practice Test - 12 - Question 11

DIRECTIONS for the question: Read the passage and answer the question based on it.

If the man to perpetuate whose memory we have this day raised a statue had been asked on what part of his busy life's work he set the highest value, he would undoubtedly have pointed to his voluminous contributions to theology. In season and out of season, he was the steadfast champion of that hypothesis respecting the Divine nature which is termed Unitarianism by its friends and Socinianism by its foes. Regardless of odds, he was ready to do battle with all comers in that cause; and if no adversaries entered the lists, he would sally forth to seek them.

To this, his highest ideal of duty, Joseph Priestley sacrificed the vulgar prizes of life, which, assuredly, were within easy reach of a man of his singular energy and varied abilities. For this object he put aside, as of secondary importance, those scientific investigations which he loved so well, and in which he showed himself so competent to enlarge the boundaries of natural knowledge and to win fame. In this cause he not only cheerfully suffered obloquy from the bigoted and the unthinking, and came within sight of martyrdom; but bore with that which is much harder to be borne than all these, the unfeigned astonishment and hardly disguised contempt of a brilliant society, composed of men whose sympathy and esteem must have been most dear to him, and to whom it was simply incomprehensible that a philosopher should seriously occupy himself with any form of Christianity.

It appears to me that the man who, setting before himself such an ideal of life, acted up to it consistently, is worthy of the deepest respect, whatever opinion may be entertained as to the real value of the tenets which he so zealously propagated and defended. But I am sure that I speak not only for myself, but for all this assemblage, when I say that our purpose to-day is to do honour, not to Priestley, the Unitarian divine, but to Priestley, the fearless defender of rational freedom in thought and in action: to Priestley, the philosophic thinker; to that Priestley who held a foremost place among "the swift runners who hand over the lamp of life," and transmit from one generation to another the fire kindled, in the childhood of the world, at the Promethean altar of Science.

The main incidents of Priestley's life are so well known that I need dwell upon them at no great length. Born in 1733, at Fieldhead, near Leeds, and brought up among Calvinists of the straitest orthodoxy, the boy's striking natural ability led to his being devoted to the profession of a minister of religion; and, in 1752, he was sent to the Dissenting Academy at Daventry--an institution which authority left undisturbed, though its existence contravened the law. The teachers under whose instruction and influence the young man came at Daventry, carried out to the letter the injunction to "try all things: hold fast that which is good," and encouraged the discussion of every imaginable proposition with complete freedom, the leading professors taking opposite sides; a discipline which, admirable as it may be from a purely scientific point of view, would seem to be calculated to make acute, rather than sound, divines. Priestley tells us, in his "Autobiography," that he generally found himself on the unorthodox side: and, as he grew older, and his faculties attained their maturity, this native tendency towards heterodoxy grew with his growth and strengthened with his strength. He passed from Calvinism to Arianism; and finally, in middle life, landed in that very broad form of Unitarianism by which his craving after a credible and consistent theory of things was satisfied.

Q. It can be said that the author of the passage is trying to:

Detailed Solution for CAT Practice Test - 12 - Question 11

One simple question for you: can you provide a one line summary for the passage? What exactly is the author of the passage doing in the passage? In this passage, he highlights the positives of Priestley and goes on to explain how he deserves to be remembered and honoured. Considering this, option 2 is the best answer in the given case.

CAT Practice Test - 12 - Question 12

DIRECTIONS for the question: Read the passage and answer the question based on it.

If the man to perpetuate whose memory we have this day raised a statue had been asked on what part of his busy life's work he set the highest value, he would undoubtedly have pointed to his voluminous contributions to theology. In season and out of season, he was the steadfast champion of that hypothesis respecting the Divine nature which is termed Unitarianism by its friends and Socinianism by its foes. Regardless of odds, he was ready to do battle with all comers in that cause; and if no adversaries entered the lists, he would sally forth to seek them.

To this, his highest ideal of duty, Joseph Priestley sacrificed the vulgar prizes of life, which, assuredly, were within easy reach of a man of his singular energy and varied abilities. For this object he put aside, as of secondary importance, those scientific investigations which he loved so well, and in which he showed himself so competent to enlarge the boundaries of natural knowledge and to win fame. In this cause he not only cheerfully suffered obloquy from the bigoted and the unthinking, and came within sight of martyrdom; but bore with that which is much harder to be borne than all these, the unfeigned astonishment and hardly disguised contempt of a brilliant society, composed of men whose sympathy and esteem must have been most dear to him, and to whom it was simply incomprehensible that a philosopher should seriously occupy himself with any form of Christianity.

It appears to me that the man who, setting before himself such an ideal of life, acted up to it consistently, is worthy of the deepest respect, whatever opinion may be entertained as to the real value of the tenets which he so zealously propagated and defended. But I am sure that I speak not only for myself, but for all this assemblage, when I say that our purpose to-day is to do honour, not to Priestley, the Unitarian divine, but to Priestley, the fearless defender of rational freedom in thought and in action: to Priestley, the philosophic thinker; to that Priestley who held a foremost place among "the swift runners who hand over the lamp of life," and transmit from one generation to another the fire kindled, in the childhood of the world, at the Promethean altar of Science.

The main incidents of Priestley's life are so well known that I need dwell upon them at no great length. Born in 1733, at Fieldhead, near Leeds, and brought up among Calvinists of the straitest orthodoxy, the boy's striking natural ability led to his being devoted to the profession of a minister of religion; and, in 1752, he was sent to the Dissenting Academy at Daventry--an institution which authority left undisturbed, though its existence contravened the law. The teachers under whose instruction and influence the young man came at Daventry, carried out to the letter the injunction to "try all things: hold fast that which is good," and encouraged the discussion of every imaginable proposition with complete freedom, the leading professors taking opposite sides; a discipline which, admirable as it may be from a purely scientific point of view, would seem to be calculated to make acute, rather than sound, divines. Priestley tells us, in his "Autobiography," that he generally found himself on the unorthodox side: and, as he grew older, and his faculties attained their maturity, this native tendency towards heterodoxy grew with his growth and strengthened with his strength. He passed from Calvinism to Arianism; and finally, in middle life, landed in that very broad form of Unitarianism by which his craving after a credible and consistent theory of things was satisfied.

Q. From the information provided in the passage, it can be inferred that Unitarianism would imply:

Detailed Solution for CAT Practice Test - 12 - Question 12

►The inference in this question has to be derived from a combination of lines extracted from the passage: In season and out of season, he was the steadfast champion of that hypothesis respecting the Divine nature which is termed Unitarianism by its friends and Socinianism by its foes and to whom it was simply incomprehensible that a philosopher should seriously occupy himself with any form of Christianity. But I am sure that I speak not only for myself, but for all this assemblage, when I say that our purpose to-day is to do honour, not to Priestley, the Unitarian divine, but to Priestley, the fearless defender of rational freedom in thought and in action.

►Unitarianism is defined as the Christian doctrine that stresses individual freedom of belief and rejects the Trinity.

►The one good thing in the question is that each option is related to Christianity and this means our answer is related to this topic as well. From the context, we can see that option 3 is the best fit in the given case.

CAT Practice Test - 12 - Question 13

DIRECTIONS for the question: Read the passage and answer the question based on it.

Citizens of the United States are quite taken with the vocabulary of liberal democracy, with words such as ‘freedom’ and ‘democracy’, which conjure key democratic values and distance the nation from the Old World taint of oligarchy and aristocracy. It is much less clear, however, that Americans are guided by democratic ideals. Or that ideology and propaganda play a crucial role in concealing the large gap between rhetoric and reality.
In truth, the Old World systems have proved extremely difficult to shrug off. In their 2014 paper, Martin Gilens and Benjamin Page argue that, as in an oligarchy, ordinary US citizens have no ‘substantial power over policy decisions [and] little or no independent influence on policy at all’.

Moreover, the US regularly subscribes to a form of managerial aristocracy. In the current presidential race, Hillary Clinton advertises her managerial expertise via the language of policy, while Donald Trump parades his via the language of business. Neither language is democratic. Neither invites self-governance.
Why is there no outcry about these oligarchical and aristocratic methods? Is it because plutocrats have power over the mechanisms of representation and repression? Is it, in short, about power? In my view, power can’t explain why voters are so enthusiastically voting for the very people who promise the least democratic outcomes. Nor are Americans knowingly rejecting democratic ideals. Instead, I see an anti-democratic ideology at work, inverting the meaning of democratic vocabulary and transforming it into propaganda.

Consider the example of mass incarceration in the US. Black Americans make up around 13 per cent of the population, but around 40 per cent of country’s ballooning prison population. Even if we assume, falsely, that black American crime rates justify this disparity, why is the state so punitive? Shouldn’t citizens instead be motivated to address the underlying socio-economic conditions that lead to such dramatic differences in behaviour between equals?

In The New Jim Crow (2010), Michelle Alexander argues that a national rhetoric of law and order has long justified mass incarceration. President Richard Nixon used it to crack down on black Americans under the cover of an epidemic of heroin use; this continued in the 1980s, as a merciless ‘war on drugs’ whose victims were all too often black men. In the US, the ideology of anti-black racism takes the view that blacks are violent and lazy, thereby masking the misapplication of the ideals of law and order.

Compare the ‘war on drugs’ to the current heroin crisis among middle-class white Americans, which has led to a national discussion of the socio-economic distress facing this class. Law and order doesn’t come into it. ‘The new face of heroin’ is new because, unlike the old face, it calls out for an empathetic response, rather than a punitive one.

But what is the flawed ideology masking the misapplication of democratic ideals? Let’s bring it out by exploring the most cherished US democratic ideal, the ideal of freedom – popularly embodied in attacks on ‘big government’. Voters are repeatedly told that ‘big government’ is the primary source of coercion that limits freedom, which it certainly sometimes does, as the Patriot Act reminds us. But corporations also limit civic freedom in significant ways.

For example, corporations are leading direct attacks on the freedom to collectively bargain. Via outsourcing, free trade agreements allow corporations to move jobs to countries where labour is cheap; meanwhile, as a result of pressure from the conservative non-profit Citizens United, corporations can fund political candidates, thereby increasing corporate control of government. The weaker a government is, the more power corporations have over it.
Voters concerned about government – as opposed to corporate – constraints on freedom are under the grip of what I will call a free market ideology. According to that ideology, the world of capital is by its nature free. All other substantial freedoms, including political freedom and personal freedom, are made possible by the freedom of markets.

Why do citizens who cherish freedom as an ideal vote to constrain their own freedoms by increasing the power of corporations? It’s because free market ideology masks the ways in which corporations deploy undemocratic modes of coercion. When a corporation bans employees from expressing, outside of work, opinions it disapproves of, this is seen as a legitimate protection of its economic interests. If workers have to sign non-disclosure contracts that silence them after they are employed elsewhere, it’s accepted as the cost of doing business.

The contradictions here are telling. If our most basic freedoms are self-expression and choiceful action, then corporations frequently limit our most basic freedoms. In liberal democratic theory, it is government that is regarded as the protector of such rights. But it’s precisely because government is attacked in the name of freedom that corporations have vastly greater power to constrain and shape it.

Q. A suitable title for the passage is:

Detailed Solution for CAT Practice Test - 12 - Question 13

The meanings of these two words:

Pervert: Change the inherent purpose or function of something/Practice sophistry; change the meaning of or be vague about in order to mislead or deceive

Adjunct: Something added to another thing but not an essential part of it

►We can clearly see that pervert fits the given negative context of the passage.

►Also, the primary subject of the passage is free market and how it is impacting democracy.
This makes option 1 the correct answer.

CAT Practice Test - 12 - Question 14

DIRECTIONS for the question: Read the passage and answer the question based on it.

Citizens of the United States are quite taken with the vocabulary of liberal democracy, with words such as ‘freedom’ and ‘democracy’, which conjure key democratic values and distance the nation from the Old World taint of oligarchy and aristocracy. It is much less clear, however, that Americans are guided by democratic ideals. Or that ideology and propaganda play a crucial role in concealing the large gap between rhetoric and reality.
In truth, the Old World systems have proved extremely difficult to shrug off. In their 2014 paper, Martin Gilens and Benjamin Page argue that, as in an oligarchy, ordinary US citizens have no ‘substantial power over policy decisions [and] little or no independent influence on policy at all’.

Moreover, the US regularly subscribes to a form of managerial aristocracy. In the current presidential race, Hillary Clinton advertises her managerial expertise via the language of policy, while Donald Trump parades his via the language of business. Neither language is democratic. Neither invites self-governance.
Why is there no outcry about these oligarchical and aristocratic methods? Is it because plutocrats have power over the mechanisms of representation and repression? Is it, in short, about power? In my view, power can’t explain why voters are so enthusiastically voting for the very people who promise the least democratic outcomes. Nor are Americans knowingly rejecting democratic ideals. Instead, I see an anti-democratic ideology at work, inverting the meaning of democratic vocabulary and transforming it into propaganda.

Consider the example of mass incarceration in the US. Black Americans make up around 13 per cent of the population, but around 40 per cent of country’s ballooning prison population. Even if we assume, falsely, that black American crime rates justify this disparity, why is the state so punitive? Shouldn’t citizens instead be motivated to address the underlying socio-economic conditions that lead to such dramatic differences in behaviour between equals?

In The New Jim Crow (2010), Michelle Alexander argues that a national rhetoric of law and order has long justified mass incarceration. President Richard Nixon used it to crack down on black Americans under the cover of an epidemic of heroin use; this continued in the 1980s, as a merciless ‘war on drugs’ whose victims were all too often black men. In the US, the ideology of anti-black racism takes the view that blacks are violent and lazy, thereby masking the misapplication of the ideals of law and order.

Compare the ‘war on drugs’ to the current heroin crisis among middle-class white Americans, which has led to a national discussion of the socio-economic distress facing this class. Law and order doesn’t come into it. ‘The new face of heroin’ is new because, unlike the old face, it calls out for an empathetic response, rather than a punitive one.

But what is the flawed ideology masking the misapplication of democratic ideals? Let’s bring it out by exploring the most cherished US democratic ideal, the ideal of freedom – popularly embodied in attacks on ‘big government’. Voters are repeatedly told that ‘big government’ is the primary source of coercion that limits freedom, which it certainly sometimes does, as the Patriot Act reminds us. But corporations also limit civic freedom in significant ways.

For example, corporations are leading direct attacks on the freedom to collectively bargain. Via outsourcing, free trade agreements allow corporations to move jobs to countries where labour is cheap; meanwhile, as a result of pressure from the conservative non-profit Citizens United, corporations can fund political candidates, thereby increasing corporate control of government. The weaker a government is, the more power corporations have over it.
Voters concerned about government – as opposed to corporate – constraints on freedom are under the grip of what I will call a free market ideology. According to that ideology, the world of capital is by its nature free. All other substantial freedoms, including political freedom and personal freedom, are made possible by the freedom of markets.

Why do citizens who cherish freedom as an ideal vote to constrain their own freedoms by increasing the power of corporations? It’s because free market ideology masks the ways in which corporations deploy undemocratic modes of coercion. When a corporation bans employees from expressing, outside of work, opinions it disapproves of, this is seen as a legitimate protection of its economic interests. If workers have to sign non-disclosure contracts that silence them after they are employed elsewhere, it’s accepted as the cost of doing business.

The contradictions here are telling. If our most basic freedoms are self-expression and choiceful action, then corporations frequently limit our most basic freedoms. In liberal democratic theory, it is government that is regarded as the protector of such rights. But it’s precisely because government is attacked in the name of freedom that corporations have vastly greater power to constrain and shape it.

Q. The author of the passage, at some point or the other in the passage, has been critical of:
I. Governments
II. Corporations
III. Political leaders

Detailed Solution for CAT Practice Test - 12 - Question 14

►The author of the passage criticizes all of the above stakeholders in the passage. Remember, the question quotes: at some point or the other. This essentially means that there has to be at least one point of criticism in the passage. One point of criticism for each of the above can be easily found. 

Criticizing political leaders:

►"In the current presidential race, Hillary Clinton advertises her managerial expertise via the language of policy, while Donald Trump parades his via the language of business. Neither language is democratic."

►"President Richard Nixon used it to crack down on black Americans under the cover of an epidemic of heroin use; this continued in the 1980s, as a merciless ‘war on drugs’ whose victims were all too often black men. In the US, the ideology of anti-black racism takes the view that blacks are violent and lazy, thereby masking the misapplication of the ideals of law and order."

►Criticizing govt "Voters are repeatedly told that ‘big government’ is the primary source of coercion that limits freedom, which it certainly sometimes does, as the Patriot Act reminds us"

►Criticizing corporations "But corporations also limit civic freedom in significant ways. 

►For example, corporations are leading direct attacks on the freedom to collectively bargain. Via outsourcing, free trade agreements allow corporations to move jobs to countries where labour is cheap; meanwhile, as a result of pressure from the conservative non-profit Citizens United, corporations can fund political candidates, thereby increasing corporate control of government.

►The weaker a government is, the more power corporations have over it."

CAT Practice Test - 12 - Question 15

DIRECTIONS for the question: Read the passage and answer the question based on it.

Citizens of the United States are quite taken with the vocabulary of liberal democracy, with words such as ‘freedom’ and ‘democracy’, which conjure key democratic values and distance the nation from the Old World taint of oligarchy and aristocracy. It is much less clear, however, that Americans are guided by democratic ideals. Or that ideology and propaganda play a crucial role in concealing the large gap between rhetoric and reality.

In truth, the Old World systems have proved extremely difficult to shrug off. In their 2014 paper, Martin Gilens and Benjamin Page argue that, as in an oligarchy, ordinary US citizens have no ‘substantial power over policy decisions [and] little or no independent influence on policy at all’.

Moreover, the US regularly subscribes to a form of managerial aristocracy. In the current presidential race, Hillary Clinton advertises her managerial expertise via the language of policy, while Donald Trump parades his via the language of business. Neither language is democratic. Neither invites self-governance.

Why is there no outcry about these oligarchical and aristocratic methods? Is it because plutocrats have power over the mechanisms of representation and repression? Is it, in short, about power? In my view, power can’t explain why voters are so enthusiastically voting for the very people who promise the least democratic outcomes. Nor are Americans knowingly rejecting democratic ideals. Instead, I see an anti-democratic ideology at work, inverting the meaning of democratic vocabulary and transforming it into propaganda.

Consider the example of mass incarceration in the US. Black Americans make up around 13 per cent of the population, but around 40 per cent of country’s ballooning prison population. Even if we assume, falsely, that black American crime rates justify this disparity, why is the state so punitive? Shouldn’t citizens instead be motivated to address the underlying socio-economic conditions that lead to such dramatic differences in behaviour between equals?

In The New Jim Crow (2010), Michelle Alexander argues that a national rhetoric of law and order has long justified mass incarceration. President Richard Nixon used it to crack down on black Americans under the cover of an epidemic of heroin use; this continued in the 1980s, as a merciless ‘war on drugs’ whose victims were all too often black men. In the US, the ideology of anti-black racism takes the view that blacks are violent and lazy, thereby masking the misapplication of the ideals of law and order.

Compare the ‘war on drugs’ to the current heroin crisis among middle-class white Americans, which has led to a national discussion of the socio-economic distress facing this class. Law and order doesn’t come into it. ‘The new face of heroin’ is new because, unlike the old face, it calls out for an empathetic response, rather than a punitive one.

But what is the flawed ideology masking the misapplication of democratic ideals? Let’s bring it out by exploring the most cherished US democratic ideal, the ideal of freedom – popularly embodied in attacks on ‘big government’. Voters are repeatedly told that ‘big government’ is the primary source of coercion that limits freedom, which it certainly sometimes does, as the Patriot Act reminds us. But corporations also limit civic freedom in significant ways.

For example, corporations are leading direct attacks on the freedom to collectively bargain. Via outsourcing, free trade agreements allow corporations to move jobs to countries where labour is cheap; meanwhile, as a result of pressure from the conservative non-profit Citizens United, corporations can fund political candidates, thereby increasing corporate control of government. The weaker a government is, the more power corporations have over it.

Voters concerned about government – as opposed to corporate – constraints on freedom are under the grip of what I will call a free market ideology. According to that ideology, the world of capital is by its nature free. All other substantial freedoms, including political freedom and personal freedom, are made possible by the freedom of markets.

Why do citizens who cherish freedom as an ideal vote to constrain their own freedoms by increasing the power of corporations? It’s because free market ideology masks the ways in which corporations deploy undemocratic modes of coercion. When a corporation bans employees from expressing, outside of work, opinions it disapproves of, this is seen as a legitimate protection of its economic interests. If workers have to sign non-disclosure contracts that silence them after they are employed elsewhere, it’s accepted as the cost of doing business.

The contradictions here are telling. If our most basic freedoms are self-expression and choiceful action, then corporations frequently limit our most basic freedoms. In liberal democratic theory, it is government that is regarded as the protector of such rights. But it’s precisely because government is attacked in the name of freedom that corporations have vastly greater power to constrain and shape it.

Q. The words 'rhetoric' and 'punitive' mean (in the respective order given):

Detailed Solution for CAT Practice Test - 12 - Question 15

►In the given case, the options are synonymous with the given words but the issue is that you need to identify the meaning in the given context.

►Here, rhetoric refers to the language designed to have a persuasive or impressive effect, but which is often regarded as lacking in sincerity or meaningful content.

Punitive refers to unfair punishment.

►Keeping these sentiments in mind, option B is the best answer in the given case.

►Penal in option B is better than burdensome in option A and crippling in option C is in context "Black Americans make up around 13 per cent of the population, but around 40 per cent of country’s ballooning prison population. Even if we assume, falsely, that black American crime rates justify this disparity, why is the state so punitive?"

CAT Practice Test - 12 - Question 16

DIRECTIONS for the question: Read the passage and answer the question based on it.

Citizens of the United States are quite taken with the vocabulary of liberal democracy, with words such as ‘freedom’ and ‘democracy’, which conjure key democratic values and distance the nation from the Old World taint of oligarchy and aristocracy. It is much less clear, however, that Americans are guided by democratic ideals. Or that ideology and propaganda play a crucial role in concealing the large gap between rhetoric and reality.

In truth, the Old World systems have proved extremely difficult to shrug off. In their 2014 paper, Martin Gilens and Benjamin Page argue that, as in an oligarchy, ordinary US citizens have no ‘substantial power over policy decisions [and] little or no independent influence on policy at all’.

Moreover, the US regularly subscribes to a form of managerial aristocracy. In the current presidential race, Hillary Clinton advertises her managerial expertise via the language of policy, while Donald Trump parades his via the language of business. Neither language is democratic. Neither invites self-governance.

Why is there no outcry about these oligarchical and aristocratic methods? Is it because plutocrats have power over the mechanisms of representation and repression? Is it, in short, about power? In my view, power can’t explain why voters are so enthusiastically voting for the very people who promise the least democratic outcomes. Nor are Americans knowingly rejecting democratic ideals. Instead, I see an anti-democratic ideology at work, inverting the meaning of democratic vocabulary and transforming it into propaganda.

Consider the example of mass incarceration in the US. Black Americans make up around 13 per cent of the population, but around 40 per cent of country’s ballooning prison population. Even if we assume, falsely, that black American crime rates justify this disparity, why is the state so punitive? Shouldn’t citizens instead be motivated to address the underlying socio-economic conditions that lead to such dramatic differences in behaviour between equals?

In The New Jim Crow (2010), Michelle Alexander argues that a national rhetoric of law and order has long justified mass incarceration. President Richard Nixon used it to crack down on black Americans under the cover of an epidemic of heroin use; this continued in the 1980s, as a merciless ‘war on drugs’ whose victims were all too often black men. In the US, the ideology of anti-black racism takes the view that blacks are violent and lazy, thereby masking the misapplication of the ideals of law and order.

Compare the ‘war on drugs’ to the current heroin crisis among middle-class white Americans, which has led to a national discussion of the socio-economic distress facing this class. Law and order doesn’t come into it. ‘The new face of heroin’ is new because, unlike the old face, it calls out for an empathetic response, rather than a punitive one.

But what is the flawed ideology masking the misapplication of democratic ideals? Let’s bring it out by exploring the most cherished US democratic ideal, the ideal of freedom – popularly embodied in attacks on ‘big government’. Voters are repeatedly told that ‘big government’ is the primary source of coercion that limits freedom, which it certainly sometimes does, as the Patriot Act reminds us. But corporations also limit civic freedom in significant ways.

For example, corporations are leading direct attacks on the freedom to collectively bargain. Via outsourcing, free trade agreements allow corporations to move jobs to countries where labour is cheap; meanwhile, as a result of pressure from the conservative non-profit Citizens United, corporations can fund political candidates, thereby increasing corporate control of government. The weaker a government is, the more power corporations have over it.

Voters concerned about government – as opposed to corporate – constraints on freedom are under the grip of what I will call a free market ideology. According to that ideology, the world of capital is by its nature free. All other substantial freedoms, including political freedom and personal freedom, are made possible by the freedom of markets.

Why do citizens who cherish freedom as an ideal vote to constrain their own freedoms by increasing the power of corporations? It’s because free market ideology masks the ways in which corporations deploy undemocratic modes of coercion. When a corporation bans employees from expressing, outside of work, opinions it disapproves of, this is seen as a legitimate protection of its economic interests. If workers have to sign non-disclosure contracts that silence them after they are employed elsewhere, it’s accepted as the cost of doing business.

The contradictions here are telling. If our most basic freedoms are self-expression and choiceful action, then corporations frequently limit our most basic freedoms. In liberal democratic theory, it is government that is regarded as the protector of such rights. But it’s precisely because government is attacked in the name of freedom that corporations have vastly greater power to constrain and shape it.

Q. According to the author of the passage, the relationship between the strength of corporations and governments is:

Detailed Solution for CAT Practice Test - 12 - Question 16

The answer can be derived from the lines: The weaker a government is, the more power corporations have over it.

CAT Practice Test - 12 - Question 17

DIRECTIONS for the question: Read the passage and answer the question based on it.

Citizens of the United States are quite taken with the vocabulary of liberal democracy, with words such as ‘freedom’ and ‘democracy’, which conjure key democratic values and distance the nation from the Old World taint of oligarchy and aristocracy. It is much less clear, however, that Americans are guided by democratic ideals. Or that ideology and propaganda play a crucial role in concealing the large gap between rhetoric and reality.

In truth, the Old World systems have proved extremely difficult to shrug off. In their 2014 paper, Martin Gilens and Benjamin Page argue that, as in an oligarchy, ordinary US citizens have no ‘substantial power over policy decisions [and] little or no independent influence on policy at all’.

Moreover, the US regularly subscribes to a form of managerial aristocracy. In the current presidential race, Hillary Clinton advertises her managerial expertise via the language of policy, while Donald Trump parades his via the language of business. Neither language is democratic. Neither invites self-governance.

Why is there no outcry about these oligarchical and aristocratic methods? Is it because plutocrats have power over the mechanisms of representation and repression? Is it, in short, about power? In my view, power can’t explain why voters are so enthusiastically voting for the very people who promise the least democratic outcomes. Nor are Americans knowingly rejecting democratic ideals. Instead, I see an anti-democratic ideology at work, inverting the meaning of democratic vocabulary and transforming it into propaganda.

Consider the example of mass incarceration in the US. Black Americans make up around 13 per cent of the population, but around 40 per cent of country’s ballooning prison population. Even if we assume, falsely, that black American crime rates justify this disparity, why is the state so punitive? Shouldn’t citizens instead be motivated to address the underlying socio-economic conditions that lead to such dramatic differences in behaviour between equals?

In The New Jim Crow (2010), Michelle Alexander argues that a national rhetoric of law and order has long justified mass incarceration. President Richard Nixon used it to crack down on black Americans under the cover of an epidemic of heroin use; this continued in the 1980s, as a merciless ‘war on drugs’ whose victims were all too often black men. In the US, the ideology of anti-black racism takes the view that blacks are violent and lazy, thereby masking the misapplication of the ideals of law and order.

Compare the ‘war on drugs’ to the current heroin crisis among middle-class white Americans, which has led to a national discussion of the socio-economic distress facing this class. Law and order doesn’t come into it. ‘The new face of heroin’ is new because, unlike the old face, it calls out for an empathetic response, rather than a punitive one.

But what is the flawed ideology masking the misapplication of democratic ideals? Let’s bring it out by exploring the most cherished US democratic ideal, the ideal of freedom – popularly embodied in attacks on ‘big government’. Voters are repeatedly told that ‘big government’ is the primary source of coercion that limits freedom, which it certainly sometimes does, as the Patriot Act reminds us. But corporations also limit civic freedom in significant ways.

For example, corporations are leading direct attacks on the freedom to collectively bargain. Via outsourcing, free trade agreements allow corporations to move jobs to countries where labour is cheap; meanwhile, as a result of pressure from the conservative non-profit Citizens United, corporations can fund political candidates, thereby increasing corporate control of government. The weaker a government is, the more power corporations have over it.

Voters concerned about government – as opposed to corporate – constraints on freedom are under the grip of what I will call a free market ideology. According to that ideology, the world of capital is by its nature free. All other substantial freedoms, including political freedom and personal freedom, are made possible by the freedom of markets.

Why do citizens who cherish freedom as an ideal vote to constrain their own freedoms by increasing the power of corporations? It’s because free market ideology masks the ways in which corporations deploy undemocratic modes of coercion. When a corporation bans employees from expressing, outside of work, opinions it disapproves of, this is seen as a legitimate protection of its economic interests. If workers have to sign non-disclosure contracts that silence them after they are employed elsewhere, it’s accepted as the cost of doing business.

The contradictions here are telling. If our most basic freedoms are self-expression and choiceful action, then corporations frequently limit our most basic freedoms. In liberal democratic theory, it is government that is regarded as the protector of such rights. But it’s precisely because government is attacked in the name of freedom that corporations have vastly greater power to constrain and shape it.

Q. According to the author of the passage:

Detailed Solution for CAT Practice Test - 12 - Question 17

In this case, you just need to understand the meanings of the given options.

►Refer to the lines: Voters concerned about government – as opposed to corporate – constraints on freedom are under the grip of what I will call a free market ideology.

►According to that ideology, the world of capital is by its nature free. All other substantial freedoms, including political freedom and personal freedom, are made possible by the freedom of markets.

►The author clearly points out how the free markets are actually impacting personal freedom. This makes option 2 the correct answer in the given case.

*Answer can only contain numeric values
CAT Practice Test - 12 - Question 18

DIRECTIONS for the question: The five sentences (labelled 1,2,3,4, and 5) given in this question, when properly sequenced, form a coherent paragraph. Decide on the proper order for the sentence and key in this sequence of five numbers as your answer.

1. The primitive man, unable to understand his being, much less the unity of all life, felt himself absolutely dependent on blind, hidden forces ever ready to mock and taunt him.
2. Again and again, the same recurrent theme -- man is nothing, the powers are everything i.e. man can bask in all the glories of the earth, but he must not become conscious of himself, an idea echoed by the State, society, and moral laws.
3. Out of that attitude grew the religious concepts of man as a mere speck of dust dependent on superior powers so high, which can only be appeased by complete surrender.
4. All the early sagas rest on that idea, which continues to be the leitmotif of the biblical tales dealing with the relation of man to God, to the State, to society.
5. The explanation of the storm raging within the individual, and between him and his surroundings, is not far to seek.


Detailed Solution for CAT Practice Test - 12 - Question 18

►5 makes for a good opening line as it introduces the topic, taken further by 1, which explains the idea.

►''that attitude'' in 3 connects with the attitude of man stated in 1. Thus 1,3 is a good, logical pairing. 

►''That idea'' in 4 continues the same logical thread, which finds a repetition in the words same, recurrent theme in 2.

*Answer can only contain numeric values
CAT Practice Test - 12 - Question 19

DIRECTIONS for the question: The five sentences (labelled 1,2,3,4, and 5) given in this question, when properly sequenced, form a coherent paragraph. Decide on the proper order for the sentence and key in this sequence of five numbers as your answer.

1. It seems doubtful that sugar-free Milanos will save us, but, maybe a new way of thinking among food researchers can.
2. The question of what to eat, when viewed through the lens of diet books and magazine weight-loss tips, can look frivolous, but in reality the stakes are high : we are raising the first generation of Americans likely to have shorter lives than their parents.
3. "For decades we've been asking the wrong question," says Gardner, associate professor of medicine at Stanford's Prevention Research Center. "It's not 'What's the best diet?' It's 'What's the best diet for each unique person?' "
4. By 2030, experts predict, obesity could be the norm, which means that the toddler squirming around in Hanover's shopping cart-who by this time had put tooth marks in the foil seal of one of the yogurt cups- is more likely than not to be obese by the time he turns 19.
5. Nutrition researcher Christopher Gardner thinks our present confusion has a lot to do with an assumption that scientists made early on: There is a single healthy diet that's right for everyone.


Detailed Solution for CAT Practice Test - 12 - Question 19

►3 and 5 are comments by the same person Gardner and we realise that 3 has to follow 5 because the 3 asks a question which is related to the point discussed in 5, that is , the assumption earlier that a single diet is right for everyone.

►The paragraph is talking about how what we eat can be very important. Sentence 2, begins with the idea and 4 continues, stating that obesity may be the norm and that sugar free Milanos will not save us, but may be a different way of looking at food.

►The 3 and 5 give the researcher Christopher Gardner''s views.

*Answer can only contain numeric values
CAT Practice Test - 12 - Question 20

DIRECTIONS for question: Four sentences related to a topic are given below. Three of them can be put together  to form a meaningful and coherent short paragraph. Identify the odd one out. Choose its number as your answer and key it in.

1. This is probably the first time in history that young readers themselves are demanding protection from the disturbing content of their course texts, yet reading has been seen as a threat to mental health for thousands of years.
2. Some contend that Virginia Woolf’s novel Mrs Dalloway (1925), in which a suicide has taken place, could trigger suicidal thoughts among those disposed to self-harm.
3. At universities around the world, students are claiming that reading books can unsettle them to the point of becoming depressed, traumatised or even suicidal.
4. Others insist that F Scott Fitzgerald’s The Great Gatsby (1925), with its undercurrent of spousal violence, might trigger painful memories of domestic abuse.
(in numerical value)


Detailed Solution for CAT Practice Test - 12 - Question 20

►In this case, the set of statements, 3-2-4, provides us with the information with regards to the nature of the paragraph.

►Statement 3 is the opening sentence and statements 2 and 4 are examples for statement 3.

►Statement 1 does not fit in the given context as it diverges from the introductory nature of the other 3 statements.

*Answer can only contain numeric values
CAT Practice Test - 12 - Question 21

DIRECTIONS for the question: The five sentences (labelled 1,2,3,4, and 5) given in this question, when properly sequenced, form a coherent paragraph. Decide on the proper order for the sentence and key in this sequence of five numbers as your answer.

1. I have bad news for you.
2. You’re an imperialist.
3. While I’m at it, let me offend you completely. Your foreign policy is an attempt “To veil the threat of terror / And check the show of pride.” You’ve vowed to “Send forth the best ye breed—Go bind your sons to exile / To serve your captives’ need.”
4. I realize that for a man like you, educated in the highest circles of modern academia, what I’ve said is a grave insult.
5. The result of all this will be to—I’ll bet you a second term—“Watch Sloth and heathen Folly / Bring all your hope to naught.”


Detailed Solution for CAT Practice Test - 12 - Question 21

►Statements 4 & 3 form a mandatory pair.

►Refer to the context in statement 4 ‘...what i’ve said is a grave insult’.

►Further, in statement 3 the author goes ahead and says ‘...let me offend you completely’, which is in continuation with author calling ‘someone’ an imperialist.

►Statement 2 would precede statements 3 & 4 as it carries the insult of being called an ‘an imperialist’.

►Statement 5 would obviously come in the end.

CAT Practice Test - 12 - Question 22

Identify the most appropriate summary for the paragraph.

Exhaustion is a vague and forgiving concept. Celebrities say they're suffering from it when they go to rehab and don't want to admit to depression or addiction. You can attribute your low mood or your short temper to exhaustion, and it can mean anything from "had a couple of bad nights' sleep" to "about to have a nervous breakdown". It also seems like a peculiarly modern affliction. Relentless email, chattering social media, never-ending images of violence and suffering in the news, the lingering effects of the financial crisis, and looming environmental catastrophe: Who's going to blame you if you confess to having had enough of it all? Anna Katharina Schaffner's Exhaustion: A History opens with the resignation of Pope Benedict XVI in 2013. He cited deteriorating physical and mental strength as a major factor in his decision to step down, and Schaffner teasingly holds him up as an emblem of our age, exhausted by the demands placed upon him.

Detailed Solution for CAT Practice Test - 12 - Question 22

►The two close answer options in this case are options 1 and 4.

Options 2 and 3 are clearly irrelevant in this case and are twist the context of the given paragraph.

Option 1 is too harsh in its nature and only focuses on the negative aspect.

►The author does wish to highlight how the term exhaustion has been abused but option 1 is too extreme in nature and therefore, ruled out.

Option 4, on the other hand, does a balanced job of representing the given condition.

*Answer can only contain numeric values
CAT Practice Test - 12 - Question 23

DIRECTIONS for question: Four sentences related to a topic are given below. Three of them can be put together  to form a meaningful and coherent short paragraph. Identify the odd one out. Choose its number as your answer and key it in.

1. This is the true synthesis of sciences. The humanistic or social sciences are supposed to take cognizance of these personal relations.
2. The physical sciences also forget their relation to man and their human origin.
3. Facts are related to our Personality and our Personalities are related to the Perfect Personality.
4. The relation between groups of facts established by persons and progressing towards the Absolute Self is the corrective principle of science.


Detailed Solution for CAT Practice Test - 12 - Question 23

►The true synthesis of science mentioned in 1 is that mentioned in 3, which is that facts and personality are related and our personalities are related to Perfect Personality.

►There is no mention in the other 3 sentences given about what is forgotten by the physical sciences.

►The word also in statement 2 indicates that there has to be a mention before.

►The missing sentence before the 2nd sentence is "The humanistic or social sciences are supposed to take cognizance of these personal relations.

*Answer can only contain numeric values
CAT Practice Test - 12 - Question 24

DIRECTIONS for the question: The five sentences (labelled 1,2,3,4, and 5) given in this question, when properly sequenced, form a coherent paragraph. Decide on the proper order for the sentence and key in this sequence of five numbers as your answer.

1. Entrepreneurs are crucial to the creation and development of every industry.
2. Amongst entrepreneurs, often the most successful and most innovative are the so-called “serial entrepreneurs,” individuals who have started multiple businesses across their careers.
3. Research from the social sciences has variously attributed the success of these individuals to risk-taking, aggression, and sociability.
4. Examining  the frequencies of three genotypes previously associated with risk-taking, aggression and social gregariousness, we find that serial entrepreneurs are distinguished more by their sociability than their aggression or risk-taking.
5. These results are the first to demonstrate a genetic basis of serial entrepreneurship, representing a new approach for understanding this class of entrepreneurs and the driving force behind their innovations.


Detailed Solution for CAT Practice Test - 12 - Question 24

►The Paragraph talks about entrepreneurs as introduced in 1 and then about serial entrepreneurs, who are often the most successful and the most innovative, as introduced in 2.

►Trying to find a link between serial entrepreneurs and abilities or characteristics and the three genotypes, sentence 3 initiated the discussion and then in 4 states the researchers found that the serial entrepreneurs are more sociable than aggressive or risk-taking.

►The sentences are in the correct sequence.

CAT Practice Test - 12 - Question 25

DIRECTIONS for the question: Identify the most appropriate summary for the paragraph.

I burst into laughter
whenever I hear
that the fish is thirsty in water.

Without the knowledge of Self
people just wander to Mathura or to Kashi
like the musk-deer unaware
of the scent in his navel,
goes on running forest to forest.

In water is the lotus plant
and the plant bears flowers
and on the flowers are the bees buzzing.
Likewise all yogis and mendicants
and all those who have renounced comforts,
are on here and hereafter and the nether world -
contemplating.

Friend, the Supreme Indestructible Being,
on whom thousands of sages meditate
and even Brahma, Vishnu and Mahesh,
really resides within one's self.

Though He is near, He appears far away -
and that is what makes one disturbed;
says Kabir, listen, O wise one,
by Guru alone is the confusion curbed.

Q. Which of the following is the best summary of the poem above?

Detailed Solution for CAT Practice Test - 12 - Question 25

►1 – is too vague

►3 -  There are two things the poet states first - that one tries to find God without when he is within, two – in order to understand this and find yourself, you need the help of the guru.

CAT Practice Test - 12 - Question 26

Read the information given below and answer the question that follows.

A multi-storeyed building has eight empty office spaces ; two on the 11th floor, two on the 12th floor, one on  the 13th floor, two on the 14th floor and one on the 15th floor. The owner of the building has to assign office space to three Insurance Companies – Tata AIG, Max New York Life and Birla Sunlife; and to three Banks – HDFC Bank, ICICI Bank and Axis Bank. Also the owner must follow the following rules in assigning office space.

1. Tata AIG may not be on the 11th floor or 15th floor.
2. No floor may remain completely unoccupied.
3. Birla Sunlife and HDFC Bank both must be on the same floor.
4. No two Insurance companies may be on the same floor.
5. Two Bank offices may not be on adjacently numbered floors.

Q. Which of the following statements could be true about the arrangement of offices in the building?

Detailed Solution for CAT Practice Test - 12 - Question 26

►The rules provided do not give enough information to allow a complete diagram of the office spaces, so the best approach is to check each of the options to see which one must be false.

►1st option is false because no two Insurance companies may be on the same floor.

►2nd option is false because HDFC Bank must be paired with Birla Sun Life and therefore, it cannot be paired with Axis Bank.

►3rd option is correct because there is not enough information to suggest that Max New York Life and Birla Sun Life may not be on adjacent floors.

►4th option is false because placing the Bank offices on the even numbered floors only would require Birla Sun Life and HDFC Bank to share one floor, and ICICI Bank and Axis Bank to share the other floor. Only two other companies remain, with three vacant floors.Since no floor may remain vacant, this arrangement is impossible.
Hence the answer is option C

CAT Practice Test - 12 - Question 27

Read the information given below and answer the question that follows.

A multi-storeyed building has eight empty office spaces ; two on the 11th floor, two on the 12th floor, one on  the 13th floor, two on the 14th floor and one on the 15th floor. The owner of the building has to assign office space to three Insurance Companies – Tata AIG, Max New York Life and Birla Sunlife; and to three Banks – HDFC Bank, ICICI Bank and Axis Bank. Also the owner must follow the following rules in assigning office space.

1. Tata AIG may not be on the 11th floor or 15th floor.
2. No floor may remain completely unoccupied.
3. Birla Sunlife and HDFC Bank both must be on the same floor.
4. No two Insurance companies may be on the same floor.
5. Two Bank offices may not be on adjacently numbered floors.
 
Q. Which of the following companies could share a floor with Axis Bank?

I. ICICI Bank
II. Max New York Life
III. Tata AIG

Detailed Solution for CAT Practice Test - 12 - Question 27

Axis Bank may not share a floor with any other office. If Axis Bank shared a floor with any other office, and considering that Birla Sun Life and HDFC Bank must share a floor, then only two other companies would remain to fill up three vacant floors and it cannot happen due to the second condition.

Hence answer is option D

CAT Practice Test - 12 - Question 28

Read the information given below and answer the question that follows.

A multi-storeyed building has eight empty office spaces ; two on the 11th floor, two on the 12th floor, one on  the 13th floor, two on the 14th floor and one on the 15th floor. The owner of the building has to assign office space to three Insurance Companies – Tata AIG, Max New York Life and Birla Sunlife; and to three Banks – HDFC Bank, ICICI Bank and Axis Bank. Also the owner must follow the following rules in assigning office space.

1. Tata AIG may not be on the 11th floor or 15th floor.
2. No floor may remain completely unoccupied.
3. Birla Sunlife and HDFC Bank both must be on the same floor.
4. No two Insurance companies may be on the same floor.
5. Two Bank offices may not be on adjacently numbered floors.

Q. If Birla Sunlife is on the 11th floor, which of the following must be false?

Detailed Solution for CAT Practice Test - 12 - Question 28

►If Birla Sun Life is on the 11th floor, then HDFC Bank must also be on the 11th floor. Because the Bank offices may not be on adjacent floors, and because the remaining floors 12 to 15 will each have exactly one company, then the Bank offices will have to be on the odd-numbered floors and the remaining two Insurance companies will have to be on the even-numbered floors.

►So 1st option, placing a bank office on an even numbered floor must be false and is the correct answer. The other answers are all possibly true.

CAT Practice Test - 12 - Question 29

Analyse the graph/s given below and answer the question that follows.

The bars in the graph below show the rate of gold (per 10 grams), the value of gold sold (in ₹ lakh) and the value of gold and silver sold (in ₹ lakh) while the line graph shows the quantity of silver sold (in kg) for the period January 2012 to December 2012.

Q. What is the average monthly growth rate of the value of gold and silver sold from January 2012 to December 2012 approximately ?

Detailed Solution for CAT Practice Test - 12 - Question 29

The value of gold and silver sold in January 2012 was around Rs. 19500 lakh and that in December 2012 was around Rs. 21000 lakh.

Over the 12-month period, the value has gone up by (21000 – 19500)/19500 = 1500/19500 = 7.6 %.

Since a 12-month period will have 11 growth periods, this is an average monthly growth of 7.6/11 = 0.69 % ≈ 0.7%.

CAT Practice Test - 12 - Question 30

Analyse the graph/s given below and answer the question that follows.

The bars in the graph below show the rate of gold (per 10 grams), the value of gold sold (in ₹ lakh) and the value of gold and silver sold (in ₹ lakh) while the line graph shows the quantity of silver sold (in kg) for the period January 2012 to December 2012.

Q. In July 2012, the rate of gold per kg was how many times the rate of silver per kg (approximately)?

Detailed Solution for CAT Practice Test - 12 - Question 30

In July 2012, the value of gold and silver sold was around Rs. 8100 lakh while the value of gold sold was around Rs. 7500 lakh.

So the value of the 2300 kg of silver sold in July 2012 was around Rs. 600 lakh.

The rate of silver was, therefore, approximately 600 × 105/2300 = Rs. 26000/kg.

The rate of gold in July 2012 was Rs. 26.3 lakh/kg.

So, the rate of gold was approximately 100 times the rate of silver.

The best answer is option 1.

View more questions
Information about CAT Practice Test - 12 Page
In this test you can find the Exam questions for CAT Practice Test - 12 solved & explained in the simplest way possible. Besides giving Questions and answers for CAT Practice Test - 12, EduRev gives you an ample number of Online tests for practice

Top Courses for CAT

Download as PDF

Top Courses for CAT